In a day there are 86,400 seconds. Write this number in scientific notation.

Answers

Answer 1
8.64x10^4 is how it’s written in scientific notation
Answer 2

Answer: 86,400 in scientific notation is 86.4* 10^3


Related Questions

The boundary line on the graph represents the equation
5x + 2y= 6.
Write an inequality that is represented by the graph.

Answers

5x+26< 6 maybe this is the answer
5x + 2y=6 that your answer

Derrick really wants to earn at least $330 at the Bike Shop. He earns $150 per week plus $15.50 for each bicycle that he assembles. How many bikes will he need to assemble this week in order to earn at least $330? (be careful when rounding in a real world problem to make sure that your answer works) Question 1 options: 10 bikes , 11 bikes , 12 bikes , 13 bikes

Answers

Answer: He will need to assemble 12 bikes, because 12 x 15.50 = $186 and $186 + 150 = $336

Step-by-step explanation: Don't need one :)) it's basic math

330-150= 180 then divide 180 by 15.50 and you will get 11.6129 and you can round that to 12 bikes

Henry divided his socks into five groups. Let s represent the total number of socks. Which expression and solution represent the number of socks in each group if s=20?

A. S/5; when s=20, the number of socks in each group is 4.


B. S/5; when s=20, the number of socks in each group is 15.


C. S-5; when s=20, the number of socks in each group is 4.


D. s-5; when s=20, the number of socks in each group is 15.

Pls help

Answers

Answer:

A

Step-by-step explanation:

If Henry has 20 socks and he split them into a group of 5, he would have 4 socks in each group. To solve this equation you must divide the number of socks by the number of groups. Once you do this, you will get the number of socks in each group.

Answer:

a

Step-by-step explanation:

Thomas decides to use the method of proportions and similar
triangles to find the height of a lamppost. He measures the length of
the post's shadow and finds it is 5 feet long. Then, he holds a 1-foot
ruler perpendicular to the ground and finds that it casts a 2-inch
shadow. How tall is the lamppost?
a. 2.5 ft c. 30 ft
b. 12 ft d. 60 ft

Answers

Answer:

The lamppost is 30 ft tall.

Step-by-step explanation:

You need to draw a diagram of the problem. Refer to attachment.

Now you need to convert the measurements that are in feet to in. The 1 foot ruler is now 12 in. and the 5 foot post's shadow is now 60 in.

The next step is to create a proportion between both angles. [tex]\frac{x}{1} =\frac{60}{2}[/tex]

x over 1 is simplified to x and 60 divided by 2 is just 30. So the lamppost is 30 ft tall.

Hope that helped. :)

I’m pretty sure it’s C

Michael decided to make French toast. He finds a recipe that calls for the following ingredients:

FRENCH TOAST
8 slices of bread
4 eggs
2 cup of milk
1 teaspoon of cinnamon

Which of the following comparison statements describe how much milk is used for the slices of bread used (There are 2 correct answers)?
A.1/4 cup of milk for every 1 slices of bread
B.2 cup of milk for every 8 slice of bread
C.1 cup of milk for every 1 slice of bread
D.1 cup of milk for every 1/8 slice of bread
E.4 cups of milk for every 1 slice of bread

Answers

The answer is B, two cups of milk for every 8 cups of bread.

This is because when using a recipe you want to make sure to get the proportions right and the ratio of milk to bread is 2:8
The answer is B for sure good luck

Please help with homework

Answers

For A the answer is 40

Sorry I could only find find for A

Could you give brainliest

Amelia needs to buy some cat food.At the nearest store,5 bags cost $28.75.How much would Amelia spend on 2 bags of cat food? Amelia would spend $ on 2 bags of cat food.

Answers

Answer:

$11.50

Step-by-step explanation:

I need help with this question.
Help needed!!!!!!!

Answers

Answer:

32

Step-by-step explanation:

Answer:

The answer is A

Step-by-step explanation:

The answer is A because the possibilities of the possibility that the answer is 'A' is just another possibility, so you just have to trust that possibility of a possibility and answer A ;)

[6 × (4 − 2) + (21 × 2/3 + 1)] ÷ 3²
If you can, please explain.

Answers

Answer:

3

Step-by-step explanation:

Use BODMAS

solve the ones in brackets first followed by multiplication/division based on the order given

Answer:

3

Step-by-step explanation:

[6 x (4 - 2) + (21 x 2/3 + 1)] ÷ 3^2

[6 x 2 + (21 x 2/3 + 1)] ÷ 3^2

21 x 2/3 = 42/3 = 14

[6 x 2 + (14 + 1)] ÷ 3^2

[6 x 2 + 15] ÷ 3^2

[12 + 15] ÷ 3^2

27 ÷ 3^2

3 x 3 = 9

27 ÷ 9 = 3

What is the difference of 16.43 – 1.043?
A 10.387
b 15.387
C 15.630

Answers

Answer:

B

Step-by-step explanation:

Just subtract

16.43-1.043=15.387

You can estimate to make it easier. :D

the answer is b - 15.387

In the town of Clover, 3 out of 5 citizens who are eligible to vote did so in the fall election. Determine the number of citizens that were eligible to vote in the fall election if 180 actually voted

Answers

Answer:

108

Step-by-step explanation:

Because 3/5(180) is 108!

Answer:

60 citizens

Step-by-step explanation:

i may be wrong but I think thsi because 180 divided by 3 = 60

Ann​'s car can go 132 miles on 4 gallons of gas. During a drive last​ weekend, she used 7 of gas. How far did she ​drive? Explain how the problem changes if you were given the distance Ann drove last weekend instead of how much gas she used.

Answers

Answer:44 Step-by-step explanation:

Answer:

multiply 132x4x7 and see how many gallons of gas

The population of a town can be modeled by f(x) = 17,200 * 1.1^x. Identify the initial value of the population. Is the population increasing or decreasing? What is the growth or decay factor? How does the population change each year?

Answers

Answer:11

Step-by-step explanation:

11

4. Is the relation represented on the graph a function? Explain

5. What is the slope of the line shown on the graph? Explain how you got your answer pls.

Answers

4. Yes because a function is a expression, rule, or law that defines a relationship between one variable and another variable which is what is happening in this image

5. Yes the slope is shown on the graph because it shows the measure of its steepness which can be determined by simply looking at the image

A recipe calls for 11 tablespoons of milk for every 16 cups of flour. If the chef puts in 80 cups of flour, how many tablespoons of milk must the chef add?

Answers

Answer:

40

Step-by-step explanation:

The answer would be 40!
Hope this helps

Mr. Rankin took his family out to dinner last night and their subtotal was $88.24. The sales tax was 5% and he left a 20% tip after the tax was added.

Answers

Answer:

$8.82

Step-by-step explanation:

Select Four answers.
2/12
1/4
2/8
6/12
4/16
3/12

Answers

Answer:

What theres no question in this?

Step-by-step explanation:

1/4 2/8 4/16 and 3/12

The marine needs 14 boats for the 42 people registered for a boat race. How many boats will they need for 6 people?​

Answers

Answer:

2

Step-by-step explanation:

We can write a proportion to determine

42 people          6 people

---------------- = ----------------

14 boats                x boats

Using cross products

42x = 6*14

42x =84

Divide each side by 42

42x/42 = 84/42

x = 2

They will need 2 boats

Plz :(
A Taxi cab costs $1.50 for the first mile and $0.75 for each additional mile. Which equation could be solved to find how many miles you can travel for $10, given that “x” is the number of additional miles? (Also solve this equation for the number of additional miles.)
A. 1.5x +0.75 = 10; x =6.17
B. 0.75x + 1.5 = 10; x = 11.333..
C. 1.5x -0.75 =10; x =7.17
D. 0.75x – 1.5 =10; x =15.33

Answers

Answer:

B

Step-by-step explanation:

I just used a calculator and did the math

it's b. 1.5 is the initial price for the first mile. then for each mile they add .75 meaning 1.5 + .75x = 10

Convert 3.25 kilograms to grams.
(1 kilogram = 1,000 grams)
Group of answer choices

0.000325

0.00325

325

3250

Answers

Answer:

3250

Step-by-step explanation;

3.25 * 1000 = 3250

A store sells a hoodie for $45, the sales tax rate in this state is 8%. What is the total price with sales tax applied?

Answers

The answer is $48.60.

The hoodie by itself has a price of $45. If the sales tax if 8% of the price you’d have to calculate the sales tax and add it to the net amount.

(45)(.08) = 3.60

(45) + 3.60 = 48.60
$48.60. multiply 45 by 0.08 to find the tax, which is $3.60. add the price plus the tax and you have the end price, which is $48.60

Enola spent $48 at a used book store where hardback books sell for $12 each and paperback books sell for $8 each.

A. Write an equation in standard form where x represents the number of hardback books and y represents the number of paperback books she bought that day:




B. If she bought 2 hardback books how many paperback books did she buy?

Answers

Answer:

A. 48= 12x + 8y

B.  3 paper back books

Step-by-step explanation:

A. If x represents hardback book, and they are $12 each then that is 12x. y = the paperback books that are $8 each, so 8y. and 48 is the amount that Enola spent.

B. 2 hardback books would equal $24.  If you had 3 paperback books that would equal $24. And the two together to get the total amount of $48.

Yes.







S
D
D
D



Be ause i know

[PICTURE ATTACHED] sigh, another variability one. if u get it correct, i’ll make you brainliest! :’)

Answers

Answer: B

Step-by-step explanation:

Option B

How many books have each of my classmates read this year?

Step-by-step explanation:

Every year the number of books each classmate read would be different

Stacy and Neil painted 280 ft2 of their living room wall with 4/5 gallon of paint. How many square feet can they paint with 4 gallons of paint?

Answers

Answer:

56

Step-by-step explanation:

Divide 280 by 4/5 and get 14 then multiply by 4 and get 56

Answer:

1400 ft2

Step-by-step explanation:

if 280 ft2 comes from 4/5 of 1 gallon of paint, then do 280 x 4, because their are 4 gallons of paint. after you x that it will be 1120 ft2. but the extra 1/5 paint from all 4 of the gallons comes out to 4/5 so add 280 ft2 to that 1120 ft2.

it comes out to 1400 ft2.

Three-sevenths of the students in a class play a sport, and, of these students, one-third play soccer. What fraction of the class plays soccer?

Answers

Answer:

I think it would be 1/7

Step-by-step explanation

Is this right? I chose 0.25. The other answers are 4, 0.5 and 2.

Answers

Answer:

its 0.25 or 2 one of those

Step-by-step explanation:

most likely 0.25 but

PLEASE HELP!!
Write an equation to represent the following statement.
51 divided by 3 is k.
Solve for k.

Answers

Answer:

51 divided by 3 is 17

Step-by-step explanation:

Hope this helps

51 divided by 3 is 17. k = 17

A math test has 12 multiplication problems and 24 division problems.

What is the ratio value of division problems to total problems? [Type your answer as a fraction in simplest form, and use / for the fraction bar.]
help help h e l p

Answers

Answer:

2 : 1

Step-by-step explanation:

24 to 12

divide by 12

2 to 1

its just simplifying

Answer:

2:1

Step-by-step explanation:

An animal gained 4 pounds steadily over 8 years. What is the unit rate of pounds per year?​

Answers

Answer:

Step-by-step explanation:

4/8 = .5 pounds/year

the answer is .5 lbs a year

This is the last one for now I promise... - w -
the question is below.

Answers

Answer: y= x +3

1) find slope using rise over run. Find your coordinates. I used (-3,0) and (0,3). Rise over run with these values : 3-0/0- -3. You should get 1 as a slope after adding and simplifying. Y=1x+b is your equation. Plug in your y and x values to the equation: 3= 0x1 +b. Multiply: 3= 0+b. B is 3( your y intercept). The equation is y=x+3

Answer:y =4,-6 hope this helps you i do my best to helps you and if it did say thanks and if you have anymore questions feel free to ask and plz rate me brainest if it did have a good night

Step-by-step explanation:

Other Questions
What mass of sulphuric acid can be made from 8g of SO3(SO3+H2O==>H2S04)09.8g18.2g 3. You have set up your Android phone using one Google account and your Android tablet using a second Google account. Now you would like to download the apps you purchased on your phone to your tablet. What is the best way to do this?a. Set up the Google account on your tablet that you used to buy apps on your phone and then download the apps.b. Buy the apps a second time from your tablet.c. Back up the apps on your phone to your SD card and then move the SD card to your tablet and transfer the apps.d. Call Google support and ask them to merge the two Google accounts into one. Which of the following statements about narratives is true? (5 points) 0 Anarrative describes events in sequence. Anarrative has characters and setting but no conflict. A narrative is a fictional account of something that has happened. A narrative must be presented in written form. Solve the following equation for I. A = 2 r + rl. Explain each step.BRAINLY IF CORRECT What function represents the amount of change given from a $10 bill, f(x), based on x, the number of bagels purchased? f(x) = 4x + O f(x) = -x + 10 Of(x) = x + 10 O f(x) = -x + 10 Explain the three types of periodicmaintanance. . can someone help me pls Given DEF : RST, find x. 12 15 16 64 Which African countries political systems are listed as in transition amid ongoing conflict? Cul es el origen de la mesa redonda Characteristics of marchantia Can we change our user names??? If you happen to deal with people whose beliefs are different from you, how are you going to explain yourself to them without being rude and without falling into logical fallacies? What will be your strategy? Which statement shows a basic difference between a democracy and an autocracy? Which of these tribes are not part of the Southeastern group? A store sells bracelets that cost x dollars and necklaces that cost y dollars. Jessica bought 4 bracelets and 7 necklaces for $18. Brianna bought 5 bracelets and 4 necklaces for $13. Gabby bought 3 bracelets and 2 necklaces. How much money did Gabby spend on bracelets and necklaces?https://brainly.com/app/ask?q=A store sells bracelets that cost x dollars and necklaces that cost y dollars. Jessica bought 4 bracelets and 7 necklaces for $18. Brianna bought 5 bracelets and 4 necklaces for $13. Gabby bought 3 bracelets and 2 necklaces. How much money did Gabby spend on bracelets and necklaces?https://brainly.com/app/ask?q=A store sells bracelets that cost x dollars and necklaces that cost y dollars. Jessica bought 4 bracelets and 7 necklaces for $18. Brianna bought 5 bracelets and 4 necklaces for $13. Gabby bought 3 bracelets and 2 necklaces. How much money did Gabby spend on bracelets and necklaces? A proportional relationship exists between x and y. When x=8 the value of y is 14. (a) Write an equation for y in terms of x. M/-4 less than or greater than 2 What does y and x equals using elimination CAN U GUYS HELP ME WITH THIS?!!! I WILL GIVE BRAINLY AND 15 POINTS!!??? THANKSKSKS!